Location via proxy:   [ UP ]  
[Report a bug]   [Manage cookies]                

2002 Cambridge Exam

Download as pdf or txt
Download as pdf or txt
You are on page 1of 22

MATHEMATICAL TRIPOS Part IA

List of Courses
Algebra and Geometry
Analysis I
Differential Equations
Dynamics
Numbers and Sets
Probability
Vector Calculus

Part IA
2

1/I/1B Algebra and Geometry


(a) State the Orbit-Stabilizer Theorem for a finite group G acting on a set X.
(b) Suppose that G is the group of rotational symmetries of a cube C. Two regular
tetrahedra T and T 0 are inscribed in C, each using half the vertices of C. What is the
order of the stabilizer in G of T ?

1/I/2D Algebra and Geometry


State the Fundamental Theorem of Algebra. Define the characteristic equation for
an arbitrary 3 × 3 matrix A whose entries are complex numbers. Explain why the matrix
must have three eigenvalues, not necessarily distinct.
Find the characteristic equation of the matrix
 
1 0 0
A =  0 0 i
0 −i 0

and hence find the three eigenvalues of A. Find a set of linearly independent eigenvectors,
specifying which eigenvector belongs to which eigenvalue.

1/II/5B Algebra and Geometry


(a) Find a subset T of the Euclidean plane R2 that is not fixed by any isometry
(rigid motion) except the identity.
Let G be a subgroup of the group of isometries of R2 , TSa subset of R2 not fixed by
any isometry except the identity, and let S denote the union g∈G g(T ). Does the group
H of isometries of S contain G? Justify your answer.
(b) Find an example of such a G and T with H 6= G.

1/II/6B Algebra and Geometry


(a) Suppose that g is a Möbius transformation, acting on the extended complex
plane. What are the possible numbers of fixed points that g can have? Justify your answer.
(b) Show that the operation c of complex conjugation, defined by c(z) = z̄, is not
a Möbius transformation.

Part IA
3

1/II/7B Algebra and Geometry


(a) Find, with justification, the matrix, with respect to the standard basis of R2 ,
of the rotation through an angle α about the origin.
(b) Find the matrix, with respect to the standard basis of R3 , of the rotation
through an angle α about the axis containing the point ( 35 , 54 , 0) and the origin. You may
express your answer in the form of a product of matrices.

1/II/8D Algebra and Geometry


Define what is meant by a vector space V over the real numbers R. Define subspace,
proper subspace, spanning set, basis, and dimension.
Define the sum U + W and intersection U ∩ W of two subspaces U and W of a
vector space V . Why is the intersection never empty?
Let V = R4 and let U = {x ∈ V : x1 − x2 + x3 − x4 = 0}, where x = (x1 , x2 , x3 , x4 ),
and let W = {x ∈ V : x1 − x2 − x3 + x4 = 0}. Show that U ∩ W has the orthogonal basis
b1 , b2 where b1 = (1, 1, 0, 0) and b2 = (0, 0, 1, 1). Extend this basis to find orthogonal
bases of U , W , and U + W . Show that U + W = V and hence verify that, in this case,

dim U + dim W = dim(U + W ) + dim(U ∩ W ) .

3/I/1A Algebra and Geometry


Given two real non-zero 2 × 2 matrices A and B, with AB = 0, show that A maps
R2 onto a line. Is it always true that BA = 0? Show that there is always a non-zero
matrix C with CA = 0 = AC. Justify your answers.

3/I/2B Algebra and Geometry


(a) What does it mean for a group to be cyclic? Give an example of a finite abelian
group that is not cyclic, and justify your assertion.
(b) Suppose that G is a finite group of rotations of R2 about the origin. Is G
necessarily cyclic? Justify your answer.

Part IA
4

3/II/5E Algebra and Geometry


Prove, using the standard formula connecting δij and ijk , that

a × (b × c) = (a · c)b − (a · b)c .

Define, in terms of the dot and cross product, the triple scalar product [a, b, c] of
three vectors a, b, c in R3 and show that it is invariant under cyclic permutation of the
vectors.
Let e1 , e2 , e3 be a not necessarily orthonormal basis for R3 , and define

e2 × e3 e3 × e1 e1 × e2
ê1 = , ê2 = , ê3 = .
[e1 , e2 , e3 ] [e1 , e2 , e3 ] [e1 , e2 , e3 ]

By calculating [ê1 , ê2 , ê3 ], show that ê1 , ê2 , ê3 is also a basis for R3 .
ˆ1 , ê
The vectors ê ˆ2 , ê
ˆ3 are constructed from ê1 , ê2 , ê3 in the same way that ê1 , ê2 , ê3
are constructed from e1 , e2 , e3 . Show that

ˆ1 = e1 , ê
ê ˆ2 = e2 , ê
ˆ 3 = e3 ,

Show that a vector V has components V · ê1 , V · ê2 , V · ê3 with respect to the basis
e1 , e2 , e3 . What are the components of the vector V with respect to the basis ê1 , ê2 , ê3 ?

3/II/6E Algebra and Geometry


(a) Give the general solution for x and y of the equations

x + y = 2a , x·y =c (c < a · a) .

Show in particular that x and y must lie at opposite ends of a diameter of a sphere whose
centre and radius should be specified.
(b) If two pairs of opposite edges of a tetrahedron are perpendicular, show that the
third pair are also perpendicular to each other. Show also that the sum of the lengths
squared of two opposite edges is the same for each pair.

Part IA
5

3/II/7A Algebra and Geometry


Explain why the number of solutions x ∈ R3 of the simultaneous linear equations
Ax = b is 0, 1 or infinite, where A is a real 3 × 3 matrix and b ∈ R3 . Let α be the mapping
which A represents. State necessary and sufficient conditions on b and α for each of these
possibilities to hold.
Let A and B be 3×3 matrices representing linear mappings α and β. Give necessary
and sufficient conditions on α and β for the existence of a 3 × 3 matrix X with AX = B.
When is X unique?
Find X when
   
4 1 1 1 1 1
A= 1
 2 1, B= 0
 1 0.
0 3 1 3 1 2

3/II/8B Algebra and Geometry


Suppose that a, b, c, d are the vertices of a regular tetrahedron T in R3 and that
a = (1, 1, 1), b = (−1, −1, 1), c = (−1, 1, −1), d = (1, x, y).
(a) Find x and y.
(b) Find a matrix M that is a rotation leaving T invariant such that M a = b and
M b = a.

Part IA
6

1/I/3C Analysis I
Suppose an ∈ R for n > 1 and a ∈ R. What does it mean to say that an → a as
n → ∞? What does it mean to say that an → ∞ as n → ∞?
Show that, if an 6= 0 for all n and an → ∞ as n → ∞, then 1/an → 0 as n → ∞.
Is the converse true? Give a proof or a counter example.
Show that, if an 6= 0 for all n and an → a with a 6= 0, then 1/an → 1/a as n → ∞.

1/I/4C Analysis I
Show that any bounded sequence of real numbers has a convergent subsequence.
Give an example of a sequence of real numbers with no convergent subsequence.
Give an example of an unbounded sequence of real numbers with a convergent
subsequence.

1/II/9C Analysis I
State some version of the fundamental axiom of analysis. State the alternating
series test and prove it from the fundamental axiom.
P∞
In each of the following cases state whether n=1 an converges or diverges and
prove your result. You may use any test for convergence provided you state it correctly.
(i) an = (−1)n (log(n + 1))−1 .
(ii) a2n = (2n)−2 , a2n−1 = −n−2 .
(iii) a3n−2 = −(2n − 1)−1 , a3n−1 = (4n − 1)−1 , a3n = (4n)−1 .
(iv) a2n +r = (−1)n (2n + r)−1 for 0 6 r 6 2n − 1, n > 0.

Part IA
7

1/II/10C Analysis I
Show that a continuous real-valued function on a closed bounded interval is bounded
and attains its bounds.
Write down examples of the following functions (no proof is required).
(i) A continuous function f1 : (0, 1) → R which is not bounded.
(ii) A continuous function f2 : (0, 1) → R which is bounded but does not attain its
bounds.
(iii) A bounded function f3 : [0, 1] → R which is not continuous.
(iv) A function f4 : [0, 1] → R which is not bounded on any interval [a, b] with
0 6 a < b 6 1.
[Hint: Consider first how to define f4 on the rationals.]

1/II/11C Analysis I
State the mean value theorem and deduce it from Rolle’s theorem.
Use the mean value theorem to show that, if h : R → R is differentiable with
0
h (x) = 0 for all x, then h is constant.
By considering the derivative of the function g given by g(x) = e−ax f (x), find all
the solutions of the differential equation f 0 (x) = af (x) where f : R → R is differentiable
and a is a fixed real number.
Show that, if f : R → R is continuous, then the function F : R → R given by
Z x
F (x) = f (t) dt
0

is differentiable with F 0 (x) = f (x).


Find the solution of the equation
Z x
g(x) = A + g(t) dt
0

where g : R → R is differentiable and A is a real number. You should explain why the
solution is unique.

Part IA
8

1/II/12C Analysis I
Prove Taylor’s theorem with some form of remainder.
An infinitely differentiable function f : R → R satisfies the differential equation

f (3) (x) = f (x)

and the conditions f (0) = 1, f 0 (0) = f 00 (0) = 0. If R > 0 and j is a positive integer,
explain why we can find an Mj such that

|f (j) (x)| 6 Mj

for all x with |x| 6 R. Explain why we can find an M such that

|f (j) (x)| 6 M

for all x with |x| 6 R and all j > 0.


Use your form of Taylor’s theorem to show that

X x3n
f (x) = .
n=0
(3n)!

Part IA
9

2/I/1D Differential Equations


Solve the equation
ÿ + ẏ − 2y = e−t
subject to the conditions y(t) = ẏ(t) = 0 at t = 0. Solve the equation

ÿ + ẏ − 2y = et

subject to the same conditions y(t) = ẏ(t) = 0 at t = 0.

2/I/2D Differential Equations


Consider the equation
1/2
1 − y2

dy
= x , (∗)
dx 1 − x2

where the positive square root is taken, within the square S : 0 6 x < 1, 0 6 y 6 1.
Find the solution that begins at x = y = 0. Sketch the corresponding solution curve,
commenting on how its tangent behaves near each extremity. By inspection of the right-
hand side of (∗), or otherwise, roughly sketch, using small line segments, the directions of
flow throughout the square S.

Part IA
10

2/II/5D Differential Equations


Explain what is meant by an integrating factor for an equation of the form

dy
+ f (x, y) = 0 .
dx

Show that 2yex is an integrating factor for

dy 2x + x2 + y 2
+ = 0,
dx 2y

and find the solution y = y(x) such that y(0) = a, for given a > 0.
Show that 2x + x2 > −1 for all x and hence that

dy 1 − y2
6 .
dx 2y

For a solution with a > 1, show graphically, by considering the sign of dy/dx first for
x = 0 and then for x < 0, that dy/dx < 0 for all x 6 0.
Sketch the solution for the case a = 1, and show that property that dy/dx → −∞
both as x → −∞ and as x → b from below, where b ≈ 0.7035 is the positive number that
satisfies b2 = e−b .
[Do not consider the range x > b.]

Part IA
11

2/II/6D Differential Equations


Solve the differential equation

dy
= ry (1 − ay)
dt
for the general initial condition y = y0 at t = 0, where r, a, and y0 are positive constants.
Deduce that the equilibria at y = a−1 and y = 0 are stable and unstable, respectively.
By using the approximate finite-difference formula

dy yn+1 − yn
=
dt δt

for the derivative of y at t = nδt, where δt is a positive constant and yn = y(n δt), show
that the differential equation when thus approximated becomes the difference equation

un+1 = λ (1 − un ) un ,

where λ = 1 + r δt > 1 and where un = λ−1 a(λ − 1) yn . Find the two equilibria and, by
linearizing the equation about them or otherwise, show that one is always unstable (given
that λ > 1) and that the other is stable or unstable according as λ < 3 or λ > 3. Show
that this last instability is oscillatory with period 2 δt. Why does this last instability have
no counterpart for the differential equation? Show graphically how this instability can
equilibrate to a periodic, finite-amplitude oscillation when λ = 3.2.

Part IA
12

2/II/7D Differential Equations


The homogeneous equation
ÿ + p(t)ẏ + q(t)y = 0
has non-constant, non-singular coefficients p(t) and q(t). Two solutions of the equation,
y(t) = y1 (t) and y(t) = y2 (t), are given. The solutions are known to be such that the
determinant
y1 y2
W (t) =
ẏ1 ẏ2
is non-zero for all t. Define what is meant by linear dependence, and show that the two
given solutions are linearly independent. Show also that
 R 
t
W (t) ∝ exp − p(s) ds .

In the corresponding inhomogeneous equation


ÿ + p(t)ẏ + q(t)y = f (t)
the right-hand side f (t) is a prescribed forcing function. Construct a particular integral
of this inhomogeneous equation in the form
y(t) = a1 (t) y1 (t) + a2 (t) y2 (t) ,
where the two functions ai (t) are to be determined such that
y1 (t) ȧ1 (t) + y2 (t) ȧ2 (t) = 0
for all t. Express your result for the functions ai (t) in terms of integrals of the functions
f (t) y1 (t)/W (t) and f (t) y2 (t)/W (t).
Consider the case in which p(t) = 0 for all t and q(t) is a positive constant, q = ω 2
say, and in which the forcing f (t) = sin(ωt). Show that in this case y1 (t) and y2 (t) can be
taken as cos(ωt) and sin(ωt) respectively. Evaluate f (t) y1 (t)/W (t) and f (t) y2 (t)/W (t)
and show that, as t → ∞, one of the ai (t) increases in magnitude like a power of t to be
determined.

2/II/8D Differential Equations


For any solution of the equations
ẋ = αx − y + y 3 (α constant)
ẏ = −x
show that
d
x2 − y 2 + 12 y 4 = 2αx2 .

dt
What does this imply about the behaviour of phase-plane trajectories at large distances
from the origin as t → ∞, in the case α = 0? Give brief reasoning but do not try to find
explicit solutions.
Analyse the properties of the critical points and sketch the phase portrait (a) in
the case α = 0, (b) in the case α = 0.1, and (c) in the case α = −0.1.

Part IA
13

4/I/3E Dynamics
The position x of the leading edge of an avalanche moving down a mountain side
making a positive angle α to the horizontal satisfies the equation
 
d dx
x = gx sin α,
dt dt

where g is the acceleration due to gravity.


By multiplying the equation by x dx
dt , obtain the first integral

2g 3
x2 ẋ2 = x sin α + c,
3
where c is an arbitrary constant of integration and the dot denotes differentiation with
respect to time.
Sketch the positive quadrant of the (x, ẋ) phase plane. Show that all solutions
approach the trajectory
 1
2g sin α 2 1
ẋ = x2 .
3
Hence show that, independent of initial conditions, the avalanche ultimately has acceler-
ation 31 g sin α.

Part IA
14

4/I/4E Dynamics
An inertial reference frame S and another reference frame S 0 have a common origin
0
O. S rotates with constant angular velocity ω with respect to S. Assuming the result
that    
da da
= +ω×a
dt S dt S 0
for an arbitrary vector a(t), show that

 d2 x   d2 x   dx 
= + 2ω × + ω × (ω × x),
dt2 S dt2 S 0 dt S 0
where x is the position vector of a point P measured from the origin.
A system of electrically charged particles, all with equal masses m and charges e,
moves under the influence of mutual central forces Fij of the form

Fij = (xi − xj )f (|xi − xj |).

In addition each particle experiences a Lorentz force due to a constant weak magnetic field
B given by
dxi
e × B.
dt

Transform the equations of motion to the rotating frame S 0 . Show that if the
angular velocity is chosen to satisfy
e
ω=− B,
2m
and if terms of second order in B are neglected, then the equations of motion in the rotating
frame are identical to those in the non-rotating frame in the absence of the magnetic field
B.

4/II/9E Dynamics
Write down the equations of motion for a system of n gravitating point particles
with masses mi and position vectors xi = xi (t), i = 1, 2, . . . , n.
Assume that xi = t2/3 ai , where the vectors ai are independent of time t. Obtain a
system of equations for the vectors ai which does not involve the time variable t.
Show that the constant vectors ai must be located at stationary points of the
function

X1 1 X X Gmi mj
mi a i · ai + .
i
9 2 j |ai − aj |
i6=j

Show that for this system, the total angular momentum about the origin and the
total momentum both vanish. What is the angular momentum about any other point?

Part IA
15

4/II/10E Dynamics
Derive the equation

d2 u f (u)
+u= ,
dθ2 mh2 u2
for the orbit r−1 = u(θ) of a particle of mass m and angular momentum hm moving under
a central force f (u) directed towards a fixed point O. Give an interpretation of h in terms
of the area swept out by a radius vector.
If the orbits are found to be circles passing through O, then deduce that the force
varies inversely as the fifth power of the distance, f = cu5 , where c is a constant. Is the
force attractive or repulsive?
Show that, for fixed mass, the radius R of the circle varies inversely as the angular
momentum of the particle, and hence that the time taken to traverse a complete circle is
proportional to R3 .
[You may assume, if you wish, the expressions for radial and transverse acceleration
in the forms r̈ − rθ̇2 , 2ṙθ̇ + rθ̈.]

4/II/11E Dynamics
An electron of mass m moving with velocity ẋ in the vicinity of the North Pole
experiences a force
x
F = aẋ × ,
|x|3
where a is a constant and the position vector x of the particle is with respect to an origin
located at the North Pole. Write down the equation of motion of the electron, neglecting
gravity. By taking the dot product of the equation with ẋ show that the speed of the
electron is constant. By taking the cross product of the equation with x show that
x
mx × ẋ − a =L,
|x|
where L is a constant vector. By taking the dot product of this equation with x, show
that the electron moves on a cone centred on the North Pole.

4/II/12E Dynamics
Calculate the moment of inertia of a uniform rod of length 2l and mass M about
an axis through its centre and perpendicular to its length. Assuming it moves in a plane,
give an expression for the kinetic energy of the rod in terms of the speed of the centre and
the angle that it makes with a fixed direction.
Two such rods are freely hinged together at one end and the other two ends slide
on a perfectly smooth horizontal floor. The rods are initially at rest and lie in a vertical
plane, each making an angle α to the horizontal. The rods subsequently move under
gravity. Calculate the speed with which the hinge strikes the ground.

Part IA
16

4/I/1C Numbers and Sets


What does it mean to say that a function f : A → B is injective? What does it
mean to say that a function g : A → B is surjective?
Consider the functions f : A → B, g : B → C and their composition g ◦ f : A → C
given by g ◦ f (a) = g(f (a)). Prove the following results.
(i) If f and g are surjective, then so is g ◦ f .
(ii) If f and g are injective, then so is g ◦ f .
(iii) If g ◦ f is injective, then so is f .
(iv) If g ◦ f is surjective, then so is g.
Give an example where g ◦ f is injective and surjective but f is not surjective and
g is not injective.

4/I/2C Numbers and Sets


If f, g : R → R are infinitely differentiable, Leibniz’s rule states that, if n > 1,

n  
dn  X n
f (x)g(x) = f (n−r) (x)g (r) (x).
dx n r
r=0

Prove this result by induction. (You should prove any results on binomial coefficients that
you need.)

4/II/5F Numbers and Sets


What is meant by saying that a set is countable?
Prove that the union of countably many countable sets is itself countable.
Let {Ji : i ∈ I} be a collection of disjoint intervals of the real line, each having
strictly positive length. Prove that the index set I is countable.

Part IA
17

4/II/6F Numbers and Sets


(a) Let S be a finite set, and let P(S) be the power set of S, that is, the set of all
subsets of S. Let f : P(S) → R be additive in the sense that f (A ∪ B) = f (A) + f (B)
whenever A ∩ B = ∅. Show that, for A1 , A2 , . . . , An ∈ P(S),
!
[ X X X
f Ai = f (Ai ) − f (Ai ∩ Aj ) + f (Ai ∩ Aj ∩ Ak )
i i i<j i<j<k
!
\
n+1
− · · · + (−1) f Ai .
i

(b) Let A1 , A2 , . . . , An be finite S


sets. Deduce from part (a) the inclusion–exclusion
formula for the size (or cardinality) of i Ai .
(c) A derangement of the set S = {1, 2, . . . , n} is a permutation π (that is, a
bijection from S to itself) in which no member of the set is fixed (that is, π(i) 6= i for
all i). Using the inclusion–exclusion formula, show that the number dn of derangements
satisfies dn /n! → e−1 as n → ∞.

4/II/7B Numbers and Sets


(a) Suppose that p is an odd prime. Find 1p + 2p + . . . + (p − 1)p modulo p.
(b) Find (p − 1)! modulo (1 + 2 + . . . + (p − 1)), when p is an odd prime.

4/II/8B Numbers and Sets


Suppose that a, b are coprime positive integers. Write down an integer d > 0 such
that ad ≡ 1 modulo b. The least such d is the order of a modulo b. Show that if the order
of a modulo b is y, and ax ≡ 1 modulo b, then y divides x.
n
Let n > 2 and Fn = 22 + 1. Suppose that p is a prime factor of Fn . Find the order
of 2 modulo p, and show that p ≡ 1 modulo 2n+1 .

Part IA
18

2/I/3F Probability
Define the indicator function IA of an event A.
Pn
Let Ii be the indicator function of the event Ai , 1 ≤ i ≤ n,
Pand let N = 1 Ii be
the number of values of i such that Ai occurs. Show that E(N ) = i pi where pi = P (Ai ),
and find var(N ) in terms of the quantities pij = P (Ai ∩ Aj ).
Using Chebyshev’s inequality or otherwise, show that

var(N )
P (N = 0) ≤ .
{E(N )}2

2/I/4F Probability
A coin shows heads with probability p on each toss. Let πn be the probability that
the number of heads after n tosses is even. Show carefully that πn+1 = (1−p)πn +p(1−πn ),
n ≥ 1, and hence find πn . [The number 0 is even.]

2/II/9F Probability
(a) Define the conditional probability P (A | B) of the event A given the event B. Let
{Bi : 1 ≤ i ≤ n} be a partition of the sample space Ω such that P (Bi ) > 0 for all i. Show
that, if P (A) > 0,
P (A | Bi )P (Bi )
P (Bi | A) = P .
j P (A | Bj )P (Bj )

(b) There are n urns, the rth of which contains r − 1 red balls and n − r blue balls. You
pick an urn (uniformly) at random and remove two balls without replacement. Find the
probability that the first ball is blue, and the conditionalPprobability that the second ball
n−1
is blue given that the first is blue. [You may assume that i=1 i(i−1) = 13 n(n−1)(n−2).]
(c) What is meant by saying that two events A and B are independent?
(d) Two fair dice are rolled. Let As be the event that the sum of the numbers shown is s,
and let Bi be the event that the first die shows i. For what values of s and i are the two
events As , Bi independent?

Part IA
19

2/II/10F Probability
There is a random number N of foreign objects in my soup, with mean µ and finite
variance. Each object is a fly with probability p, and otherwise is a spider; different objects
have independent types. Let F be the number of flies and S the number of spiders.
(a) Show that GF (s) = GN (ps + 1 − p). [GX denotes the probability generating function
of a random variable X. You should present a clear statement of any general result used.]
(b) Suppose N has the Poisson distribution with parameter µ. Show that F has the
Poisson distribution with parameter µp, and that F and S are independent.
(c) Let p = 12 and suppose that F and S are independent. [You are given nothing about
the distribution of N .] Show that GN (s) = GN ( 21 (1 + s))2 . By working with the function
H(s) = GN (1 − s) or otherwise,
n deduce that N has the Poisson distribution. [You may
assume that 1 + nx + o(n−1 ) → ex as n → ∞.]

2/II/11F Probability
Let X, Y , Z be independent random variables each with the uniform distribution
on the interval [0, 1].
(a) Show that X + Y has density function
u if 0 ≤ u ≤ 1,
(
fX+Y (u) = 2 − u if 1 ≤ u ≤ 2,
0 otherwise.

(b) Show that P (Z > X + Y ) = 61 .


(c) You are provided with three rods of respective lengths X, Y , Z. Show that the
probability that these rods may be used to form the sides of a triangle is 12 .
(d) Find the density function fX+Y +Z (s) of X + Y + Z for 0 6 s 6 1. Let W be uniformly
distributed on [0, 1], and independent of X, Y , Z. Show that the probability that rods of
lengths W , X, Y , Z may be used to form the sides of a quadrilateral is 56 .

2/II/12F Probability
(a) Explain what is meant by the term ‘branching process’.
(b) Let Xn be the size of the nth generation of a branching process in which each family
size has probability generating function G, and assume that X0 = 1. Show that the
probability generating function Gn of Xn satisfies Gn+1 (s) = Gn (G(s)) for n ≥ 1.
(c) Show that G(s) = 1 − α(1 − s)β is the probability generating function of a non-negative
integer-valued random variable when α, β ∈ (0, 1), and find Gn explicitly when G is thus
given.
(d) Find the probability that Xn = 0, and show that it converges as n → ∞ to 1−α1/(1−β) .
Explain carefully why this implies that the probability of ultimate extinction equals
1 − α1/(1−β) .

Part IA
20

3/I/3A Vector Calculus


Determine whether each of the following is the exact differential of a function, and
if so, find such a function:
(a) (cosh θ + sinh θ cos φ)dθ + (cosh θ sin φ + cos φ)dφ,
(b) 3x2 (y 2 + 1)dx + 2(yx3 − z 2 )dy − 4yzdz.

3/I/4A Vector Calculus


State the divergence theorem.
Consider the integral Z
I= rn r · dS ,
S

where n > 0 and S is the sphere of radius R centred at the origin. Evaluate I directly,
and by means of the divergence theorem.

3/II/9A Vector Calculus


Two independent variables x1 and x2 are related to a third variable t by

x1 = a + αt , x2 = b + βt ,

where a, b, α and β are constants. Let f be a smooth function of x1 and x2 , and let
F (t) = f (x1 , x2 ). Show, by using the Taylor series for F (t) about t = 0, that

∂f ∂f
f (x1 , x2 ) = f (a, b) + (x1 − a) + (x2 − b)
∂x1 ∂x2
2 2 2
 
1 2 ∂ f ∂ f 2∂ f
+ (x1 − a) + 2(x1 − a)(x2 − b) + (x2 − b) + ...,
2 ∂x21 ∂x1 ∂x2 ∂x22

where all derivatives are evaluated at x1 = a, x2 = b.


Hence show that a stationary point (a, b) of f (x1 , x2 ) is a local minimum if

H11 > 0, det Hij > 0 ,

∂2f
where Hij = ∂xi ∂xj is the Hessian matrix evaluated at (a, b).
Find two local minima of

f (x1 , x2 ) = x41 − x21 + 2x1 x2 + x22 .

Part IA
21

3/II/10A Vector Calculus


The domain S in the (x, y) plane is bounded by y = x, y = ax (0 6 a 6 1) and
xy 2 = 1 (x, y > 0). Find a transformation

u = f (x, y), v = g(x, y) ,

such that S is transformed into a rectangle in the (u, v) plane.


Evaluate
y2 z2
Z
dx dy dz ,
D x
where D is the region bounded by

y = x, y = zx, xy 2 = 1 (x, y > 0)

and the planes


z = 0, z = 1.

3/II/11A Vector Calculus


Prove that

∇ × (a × b) = a∇ · b − b∇ · a + (b · ∇)a − (a · ∇)b.

S is an open orientable surface in R3 with unit normal n, and v(x) is any


continuously differentiable vector field such that n · v = 0 on S. Let m be a continuously
differentiable unit vector field which coincides with n on S. By applying Stokes’ theorem
to m × v, show that Z I
 ∂vi
δij − ni nj dS = u · vds ,
S ∂xj C

where s denotes arc-length along the boundary C of S, and u is such that uds = ds × n.
Verify this result by taking v = r, and S to be the disc |r| 6 R in the z = 0 plane.

Part IA
22

3/II/12A Vector Calculus


(a) Show, using Cartesian coordinates, that ψ = 1/r satisfies Laplace’s equation,
∇2 ψ = 0, on R3 \{0}.
(b) φ and ψ are smooth functions defined in a 3-dimensional domain V bounded
by a smooth surface S. Show that
Z Z
(φ∇2 ψ − ψ∇2 φ)dV = (φ∇ψ − ψ∇φ) · dS.
V S

(c) Let ψ = 1/|r − r0 | , and let Vε be a domain bounded by a smooth outer surface
S and an inner surface Sε , where Sε is a sphere of radius ε, centre r0 . The function φ
satisfies
∇2 φ = −ρ(r).
Use parts (a) and (b) to show, taking the limit ε → 0, that φ at r0 is given by
Z Z  
ρ(r) 1 ∂φ ∂ 1
4πφ(r0 ) = dV + − φ(r) dS,
V |r − r0 | S |r − r0 | ∂n ∂n |r − r0 |

where V is the domain bounded by S.

Part IA

You might also like